6 Christoffel の記号を用いたdivergence,Laplacianの導出

発散を求めるときに現れる $ \Gamma_{ij}^k$ に関しては、 便利な公式が知られている。 例えば、正則な行列 $ A=\left(a_{ij}\right)$ とその逆行列 $ A^{-1}=B = \left(b^{ij}\right)$ を考える。 行列 $ A$ の行列式を $ \det A$ と書くことにする。 また行列 $ A$ の要素 $ a_{ij}$ に対する余因子(cofactor) $ \Delta^{ij}$ と書くとすれば、 行列 $ A$ の逆行列 $ B$ の要素 $ b^{ij}$

$\displaystyle b^{ij}= {\Delta^{ji}}/{\det A}$ (54)

で与えられることが知られている。 ここで、余因子は行列 $ A$からその $ i$$ j$ 列を取り除いた小行列 $ \tilde{A}_{ij}$ を使って

$\displaystyle \Delta^{ij} =(-1)^{i+j} \det \tilde{A}_{ij}
$

で定義される。 $ \delta_i^k = \sum_{j} a_{ij}b^{jk} =\sum_{j} a_{ij} \Delta^{kj}/\det A$ であるから、 $ i=k$ として

$\displaystyle \sum_{j} a_{ij} \Delta^{ij} =\det A$ (55)

を得る。 ここで $ i$ については和をとらない。 定義より $ \Delta^{ij}$ に要素 $ a^{ij}$ は含まれないので

$\displaystyle \frac{\partial}{\partial a_{ij}} \left(\det A \right) = \Delta^{ij}$ (56)

が得られる。 これを使えば

$\displaystyle \frac{\partial}{\partial x^k} \left(\det A \right) = \sum_{ij} \f...
...(\det A \right) \mathrm{Tr} \left(A^{-1} \frac{\partial A}{\partial x^k}\right)$ (57)

従って

$\displaystyle \mathrm{Tr} \left(A^{-1} \frac{\partial A}{\partial x^k}\right)= \frac{\partial}{\partial x^k} \left(\ln \det A\right)$ (58)

が得られる。 ここで $ \mathrm{Tr}  M$ は行列 $ M$ の対角成分の和を表す。

$ \Gamma_{ij}^j$

$\displaystyle \Gamma_{ij}^j= \frac{1}{2} g^{kj} \left(
\frac{\partial g_{ik}}{\...
...c{\partial g_{jk}}{\partial x^i}
-\frac{\partial g_{ij}}{\partial x^k}
\right)
$

であるから、零以外の値をとるには $ k=j$ でなければならないので、これを踏まえると (58) を用いて

$\displaystyle \Gamma_{ij}^j$ $\displaystyle = \frac{1}{2}g^{jk} \frac{\partial g_{kj}}{\partial x^i} +\frac{1...
...}}{\partial x^m}\right) =\frac{1}{2}g^{jk} \frac{\partial g_{kj}}{\partial x^i}$    
  $\displaystyle = \frac{1}{2} \frac{\partial}{\partial x^i} \left(\ln g \right) =...
...ft(\ln g^{1/2}\right) = \frac{\partial}{\partial x^i} \left(\ln \sqrt{g}\right)$    
  $\displaystyle =\frac{1}{\sqrt{g}} \frac{\partial }{\partial x^i} \sqrt{g}$ (59)

となることが分かる。 ここで $ g=\det g_{ij}$ である。 これから

$\displaystyle \mathbf{\nabla}\cdot \vec{\xi} \equiv \xi_{;j}^j =\frac{\partial ...
...frac{1}{\sqrt{g}} \frac{\partial}{\partial x^j} \left( \sqrt{g}   \xi^j\right)$ (60)

を得る。同様に

$\displaystyle \mathbf{\nabla}^2 \Phi \equiv \left(g^{ij} \frac{\partial \Phi}{\...
...partial x^i} \left( \sqrt{g}  g^{ij} \frac{\partial \Phi}{\partial x^j}\right)$ (61)

を得るが、これを展開すると

$\displaystyle \frac{1}{\sqrt{g}} \frac{\partial}{\partial x^i} \left( \sqrt{g}  g^{ij} \frac{\partial \Phi}{\partial x^j}\right)$ $\displaystyle = \frac{g^{ij}}{\sqrt{g}}  \left( \frac{\partial}{\partial x^i} ...
...{\partial}{\partial x^i} \left(g^{ij} \frac{\partial \Phi}{\partial x^j}\right)$    
  % latex2html id marker 5225
$\displaystyle =\frac{\partial}{\partial x^i} \left(...
...^j}\right) +\Gamma_{ki}^i g^{kj} \frac{\partial \Phi}{\partial x^j} =(\ref{40})$    

であるから、(43) で計算したものと一致していることが分かる。

fat-cat 平成16年11月29日